6

Let $n \in \mathbb{N}$. If $x>0$ is such that $x^{n}+\frac{1}{x^n}$ and $x^{n+1}+\frac{1}{x^{n+1}}\in \mathbb{Q} \implies x+\frac{1}{x}\in\mathbb{Q}$?

Any thoughts on how to solve the above problem. Working for $n=2$ says that this result is true, but not sure if one can generalize

Axler
  • 61
  • is it for any $n$ or just some? – Zelos Malum Mar 08 '16 at 06:54
  • 3
    You have used the formula $$x^{n+1}+\frac {1}{x^{n+1}}=\left(x^n+\frac{1}{x^n} \right)\left(x+ \frac 1x \right)-\left(x^{n-1}+\frac{1}{x^{n-1}} \right)?$$ – Roman83 Mar 08 '16 at 07:24
  • Related : http://math.stackexchange.com/questions/1618334/use-induction-to-prove-that-for-all-n-in-mathbbn-xn-frac1xn-in?lq=1 and http://math.stackexchange.com/questions/936479/proving-that-frac-phi4001-phi200-is-an-integer – lab bhattacharjee Mar 08 '16 at 08:29
  • Related https://math.stackexchange.com/questions/1827466/s-n-is-an-integer-for-all-integers-n – rtybase Jun 24 '18 at 11:41
  • A special case: If $p=x^4+\dfrac1{x^4}$ and $q=x^5+\dfrac1{x^5}$, then $$x+\frac1x=\frac{p^2q-pq-q}{p^3-2p-q^2+1}$$ – Akiva Weinberger Jan 25 '24 at 09:50

3 Answers3

6

Proffering the following argument based on elementary properties of algebraic numbers.

From $x^n+x^{-n}=q_1$ it follows that $x$ satisfies the polynomial equation $x^{2n}-q_1x^n+1=0$. Furthermore, it is obvious that all the zeros of this polynomial are $x^{\epsilon}\zeta_n^k$, where $\epsilon=\pm1, \zeta_n=e^{2\pi i/n}$ and $k=0,1,2,\ldots,n-1$.

Therefore the zeros of the minimal polynomial $m(T)$ of $x$ (over $\Bbb{Q}$) are among those numbers.

But, from $x^{n+1}+x^{-(n+1)}=q_2$ it similarly follows that the zeros of the minimal polynomial of $x$ are among the numbers $x^{\epsilon}\zeta_{n+1}^\ell, \ell=0,1,2,\ldots,n$.

Therefore the zeros of $m(T)$ are either just $x$, or both $x$ and $x^{-1}$. In the former case $x$ is rational, and the claim is immediate. In the latter case $x+\dfrac1x$ is rational because it is the coefficient of the linear term of the minimal polynomial $m(T)=(T-x)(T-1/x)\in\Bbb{Q}[x]$.


This argument also proves that if $x^n+x^{-n}$ and $x^{n+1}+x^{-(n+1)}$ are both integers, then $x+1/x$ must also be an integer. This is because in this case $x$ is an algebraic integer, and hence the coefficients of $m(T)$ are all integers. This old trick then implies that $x^k+x^{-k}\in\Bbb{Z}$ for all $k\in\Bbb{Z}$.

Jyrki Lahtonen
  • 133,153
  • Can this approach also be applied to https://math.stackexchange.com/q/4533243/42969? – Martin R Sep 17 '22 at 11:18
  • @MartinR Yes, the same approach works, when rationality (or integrality) of $x^k+x^{-k}$ is known for two coprime choices of $k$. The intersection of the sets of roots of the two resulting equations can only have $x$ and $1/x$. – Jyrki Lahtonen Sep 17 '22 at 13:38
  • Would that be worth another answer to the above question? – Martin R Sep 17 '22 at 13:53
4

The idea is to write $x+\frac{1}{x}$ in terms of $x^n+\frac{1}{x^n}$ and $x^{n+1}+\frac{1}{x^{n+1}}$. A domino-sum does the job here.

Put $w_n=x^n+\frac{1}{x^n}$. Then, the fundamental identity is

$$ w_{i}w_{j}=w_{i-j}+w_{i+j} \ (i,j\in{\mathbb Z}) \tag{1} $$

Say that a number $i$ is nice when $w_i\in{\mathbb Q}$. Note that $0$ is nice since $w_0=2$. It follows then from (1) (with $j=i$) that if $i$ is nice then $2i$ is nice also. Using (1) again (with $j=2i$) we see that $3i$ is nice also. More generally, by induction we have that any multiple of a nice integer is nice.

Next, look at those identities : (they all follow from (1))

$$ \begin{array}{lclclclcl} w_{n+1}w_{n} &=& w_1 &+& w_{2n+1} & & & & \\ w_{n+1}w_{3n} &=& & & w_{2n-1} &+& w_{4n+1} & & \\ w_{n+1}w_{5n} &=& & & & & w_{4n-1} &+& w_{6n+1} \\ \end{array}\tag{2} $$

If we look at terms in the same column, the sum is a multiple of $w_1$ : $w_{2n+1}+w_{2n-1}=w_1w_{2n}$, $w_{4n+1}+w_{4n-1}=w_1w_{4n}$ etc. Formally, we have for any $r>0$,

$$ w_{n+1}\bigg(\sum_{j=1}^r w_{(2j-1)n}\bigg)= w_1+\sum_{j=1}^{r-1} w_1w_{2jn} +w_{2rn+1} \tag{3} $$

We can rewrite this as

$$ w_1=\frac{w_{n+1}\bigg(\sum_{j=1}^r w_{(2j-1)n}\bigg)-w_{2rn+1}}{1+\sum_{j=1}^{r-1} w_{2jn}} \tag{4} $$

In the RHS of (4), all the variables are rational numbers, except possibly $w_{2rn+1}$. When $n$ is even, for suitable $r$ (for example $r=\frac{n+1}{2}$), $2rn+1$ will be divisible by $n+1$, so $w_{2rn+1}$ is rational also, which finishes the proof (note that the denominator is nonzero because it is positive). Similarly, when $n$ is even one can use the identity

$$ w_1=\frac{w_{n+1}\bigg(1+\sum_{j=2}^r w_{2jn}\bigg)-w_{2rn+n+1}}{1+\sum_{j=2}^{r} w_{(2j-1)n}} \tag{4'} $$

and take a suitable value of $r$ so that $2rn$ is a multiple of $n+1$. To derive (4'), notice that $d_j=w_{n+1}w_{2jn}-w_1w_{(2j-1)n}$ can be simplified to $d_j=u_{j}-u_{j-1}$ where $u_{j}=w_{2jn+n+1}$. Then $\sum_{j=1}^r d_j = u_{r}-u_{0}$, and (4') follows.

Ewan Delanoy
  • 61,600
  • Do you mean when $n$ is even then $r=\frac{n+2}{2}$ factorizes $2rn+1$ with respect to $n+1$? Because in your example $n^2+n+1$ is not a multiple of $n+1$ – Diger Jun 23 '18 at 23:33
  • Hm...$r=\frac{k(n+1)-1}{2n}$ has to be an integer. $k$ even doesn't work as then the nominator would be odd. Therefore $$ r=\frac{(2k+1)(n+1)-1}{2n} = \frac{2kn+2k+n}{2n} $$ has to be an integer. For odd $n$ the second and third term will however be odd and thus it only works for even $n$? – Diger Jun 24 '18 at 00:25
  • @Diger please see my update. – Ewan Delanoy Jun 24 '18 at 11:31
  • Can this approach also be applied to https://math.stackexchange.com/q/4533243/42969? – Martin R Sep 17 '22 at 11:18
-1

Well, if $x^n\in\mathbb{Q}$, then $\frac{1}{x^n}\in\mathbb{Q}$, so we can ask the question if $x^n\in\mathbb{Q}$ for all $m$ in $n+m$ if $n$ and $n+1$ is rational, and any rational number to the power of a natural number is indeed rational, and no irrational number can be written as both $\sqrt[2]{a/b}$ and $\sqrt[3]{a/b}$, (so for example $\sqrt{2}$ is rational if squared, but not if cubed), so yes, $x+\frac{1}{x}\in\mathbb{Q}$ if $x^n+\frac{1}{x^n}\in\mathbb{Q}$

Tommy R
  • 102
  • 6